Doubt

This topic has expert replies
User avatar
Master | Next Rank: 500 Posts
Posts: 269
Joined: Sun Apr 27, 2014 10:33 pm
Thanked: 8 times
Followed by:5 members

Doubt

by prachi18oct » Thu Jun 18, 2015 12:17 pm
A telecommunications company's stock fell 10% last month, while a rival company's stock increased by 10%. The rival company's stock value today is what percentage of the telecommunication company's stock value at the beginning of last month?

(1) The rival company's stock was valued at $42 per share at the beginning of last month.

(2) The dollar amount of the rival company's stock increase was 90% as much as the dollar amount of the decrease in the telecommunication company's stock.

A)Statement (1) ALONE is sufficient, but statement (2) alone is not sufficient to answer the question asked
B) Statement (2) ALONE is sufficient, but statement (1) alone is not sufficient to answer the question asked
C) Both statements (1) and (2) TOGETHER are sufficient to answer the question asked; but NEITHER statement ALONE is sufficient
D) EACH statement ALONE is sufficient to answer the question asked
E) Statements (1) and (2) TOGETHER are NOT sufficient to answer the question asked, and additional data specific to the problem are needed


Is the dollar amount increase is the amount increased in the share value?
For e.g : if the share value = 5 and new share value = 8 then dollar amount increase in stock is 3 ?

User avatar
GMAT Instructor
Posts: 15539
Joined: Tue May 25, 2010 12:04 pm
Location: New York, NY
Thanked: 13060 times
Followed by:1906 members
GMAT Score:790

by GMATGuruNY » Thu Jun 18, 2015 12:56 pm
prachi18oct wrote:A telecommunications company's stock fell 10% last month, while a rival company's stock increased by 10%. The rival company's stock value today is what percentage of the telecommunication company's stock value at the beginning of last month?

(1) The rival company's stock was valued at $42 per share at the beginning of last month.

(2) The dollar amount of the rival company's stock increase was 90% as much as the dollar amount of the decrease in the telecommunication company's stock.
The question stem asks for the following ratio:
(rival's price today)/(telecom's original price).

Statement 1: The rival company's stock was valued at $42 per share at the beginning of last month.
After the 10% increase, the rival's price today = 42 + 10% of 42 = 46.2.
If the telecom's original price = 1, then (rival's price today)/(telecom's original price) = 46.2/1 = 46.2.
If the telecom's original price = 10, then (rival's price today)/(telecom's original price) = 46.2/10 = 4.62.
Since the ratio can be different values, INSUFFICIENT.

Statement 2: The dollar amount of the rival company's stock increase was 90% as much as the dollar amount of the decrease in the telecommunication company's stock.
Case 1: rival's increase = 9, telecom's decrease = 10

Since the rival's $9 increase is equal to 10% of the rival's original price -- and 9 is 10% of 90 -- the rival's original price = 90.
Thus, the rival's price today = 90+9 = 99.

Since the telecom's $10 decrease is equal to 10% of the telecom's original price -- and 10 is 10% of 100 -- the telecom's original price = 100.

Resulting ratio:
(rival's price today)/(telecom's original price) = 99/100.

Case 2: rival's increase = 90, telecom's decrease = 100
Since the rival's $90 increase is equal to 10% of the rival's original price -- and 90 is 10% of 900 -- the rival's original price = 900.
Thus, the rival's price today = 900+90 = 990.

Since the telecom's decrease of $100 is equal to 10% of the telecom's original price -- and 100 is 10% of 1000 -- the telecom's original price = 1000.
Resulting ratio:
(rival's price today)/(telecom's original price) = 990/1000 = 99/100.

Since the ratio is THE SAME in each case, SUFFICIENT.

The correct answer is B.
Private tutor exclusively for the GMAT and GRE, with over 20 years of experience.
Followed here and elsewhere by over 1900 test-takers.
I have worked with students based in the US, Australia, Taiwan, China, Tajikistan, Kuwait, Saudi Arabia -- a long list of countries.
My students have been admitted to HBS, CBS, Tuck, Yale, Stern, Fuqua -- a long list of top programs.

As a tutor, I don't simply teach you how I would approach problems.
I unlock the best way for YOU to solve problems.

For more information, please email me (Mitch Hunt) at [email protected].
Student Review #1
Student Review #2
Student Review #3

GMAT/MBA Expert

User avatar
GMAT Instructor
Posts: 2621
Joined: Mon Jun 02, 2008 3:17 am
Location: Montreal
Thanked: 1090 times
Followed by:355 members
GMAT Score:780

by Ian Stewart » Thu Jun 18, 2015 1:21 pm
prachi18oct wrote:A telecommunications company's stock fell 10% last month, while a rival company's stock increased by 10%. The rival company's stock value today is what percentage of the telecommunication company's stock value at the beginning of last month?

(1) The rival company's stock was valued at $42 per share at the beginning of last month.

(2) The dollar amount of the rival company's stock increase was 90% as much as the dollar amount of the decrease in the telecommunication company's stock.
If we let c and r be, respectively, the value of the company and rival stock last month, we know they changed as follows:

company: c ---> 0.9c
rival: r ---> 1.1r

The question asks us to find 1.1r/c, or 1.1(r/c), so if we can find r/c, we can answer the question.

Statement 1 clearly is insufficient since we have no information about c.

We know the rival's increase was 0.1r, and the company's decrease was 0.1c. From Statement 2, we know 0.1r is 90% of 0.1c, so we know that 0.1r = (0.9)(0.1c), so r/c = 0.9. Since we only need to know r/c to answer the question, S2 is sufficient and the answer is B (if you actually want to answer the question, we find that 1.1(r/c) = 1.1(0.9) = 0.99). .
For online GMAT math tutoring, or to buy my higher-level Quant books and problem sets, contact me at ianstewartgmat at gmail.com

ianstewartgmat.com

Legendary Member
Posts: 518
Joined: Tue May 12, 2015 8:25 pm
Thanked: 10 times

by nikhilgmat31 » Fri Jun 26, 2015 3:22 am
Answer is B.

90 %

GMAT Instructor
Posts: 2630
Joined: Wed Sep 12, 2012 3:32 pm
Location: East Bay all the way
Thanked: 625 times
Followed by:119 members
GMAT Score:780

by Matt@VeritasPrep » Mon Jun 29, 2015 4:27 pm
Last month:

Our company = x
Rival = y

This month:

Our company = .9x
Rival = 1.1y

We want 1.1y / x.

S1 tells us y = 42. Doesn't give x, not sufficient.
S2 tells us .1y = .9*(.1x). This gives us x/y = .1/(.9*.1), and 1.1x/y = 1.1 * (.1/(.9*.1)). That's a value -- don't bother computing it -- so S2 is sufficient.

Legendary Member
Posts: 518
Joined: Tue May 12, 2015 8:25 pm
Thanked: 10 times

by nikhilgmat31 » Mon Jun 29, 2015 10:59 pm
Answer is B

c is cost of share of first company
r is cost of share of rival company

10/100 r = 90/100 * 10/100 * c

r/c = 90/100